Suppose $sum_{n=1}^{infty}sqrt{{a_{n}}/{n}}$ is convergent. Prove that $sum_{n=1}^{infty}a_{n}$ is also...












6












$begingroup$


Let ${a_{n}}$ be a decreasing sequence of non-negative real numbers. Suppose $sum_{n=1}^{infty}sqrt{frac{a_{n}}{n}}$ is convergent. Prove that $sum_{n=1}^{infty}a_{n}$ is also convergent.
My thought is to use the direct comparison test but I think I'm struggling with showing that $a_{n}leqsqrt{frac{a_{n}}{n}}$ $forall$ n $inmathbb{N}$.
Any help would be great. Thank you!










share|cite|improve this question











$endgroup$








  • 5




    $begingroup$
    It is notable that $sqrt{a_n/n}$ is the geometric mean of $a_n$ and $frac 1n$
    $endgroup$
    – Omnomnomnom
    Jan 2 at 3:58










  • $begingroup$
    Why the downvote and the vote to close?
    $endgroup$
    – JavaMan
    Jan 2 at 4:19


















6












$begingroup$


Let ${a_{n}}$ be a decreasing sequence of non-negative real numbers. Suppose $sum_{n=1}^{infty}sqrt{frac{a_{n}}{n}}$ is convergent. Prove that $sum_{n=1}^{infty}a_{n}$ is also convergent.
My thought is to use the direct comparison test but I think I'm struggling with showing that $a_{n}leqsqrt{frac{a_{n}}{n}}$ $forall$ n $inmathbb{N}$.
Any help would be great. Thank you!










share|cite|improve this question











$endgroup$








  • 5




    $begingroup$
    It is notable that $sqrt{a_n/n}$ is the geometric mean of $a_n$ and $frac 1n$
    $endgroup$
    – Omnomnomnom
    Jan 2 at 3:58










  • $begingroup$
    Why the downvote and the vote to close?
    $endgroup$
    – JavaMan
    Jan 2 at 4:19
















6












6








6


7



$begingroup$


Let ${a_{n}}$ be a decreasing sequence of non-negative real numbers. Suppose $sum_{n=1}^{infty}sqrt{frac{a_{n}}{n}}$ is convergent. Prove that $sum_{n=1}^{infty}a_{n}$ is also convergent.
My thought is to use the direct comparison test but I think I'm struggling with showing that $a_{n}leqsqrt{frac{a_{n}}{n}}$ $forall$ n $inmathbb{N}$.
Any help would be great. Thank you!










share|cite|improve this question











$endgroup$




Let ${a_{n}}$ be a decreasing sequence of non-negative real numbers. Suppose $sum_{n=1}^{infty}sqrt{frac{a_{n}}{n}}$ is convergent. Prove that $sum_{n=1}^{infty}a_{n}$ is also convergent.
My thought is to use the direct comparison test but I think I'm struggling with showing that $a_{n}leqsqrt{frac{a_{n}}{n}}$ $forall$ n $inmathbb{N}$.
Any help would be great. Thank you!







real-analysis calculus convergence






share|cite|improve this question















share|cite|improve this question













share|cite|improve this question




share|cite|improve this question








edited Jan 2 at 10:07









user1551

74.6k566129




74.6k566129










asked Jan 2 at 3:46









dorkichardorkichar

1024




1024








  • 5




    $begingroup$
    It is notable that $sqrt{a_n/n}$ is the geometric mean of $a_n$ and $frac 1n$
    $endgroup$
    – Omnomnomnom
    Jan 2 at 3:58










  • $begingroup$
    Why the downvote and the vote to close?
    $endgroup$
    – JavaMan
    Jan 2 at 4:19
















  • 5




    $begingroup$
    It is notable that $sqrt{a_n/n}$ is the geometric mean of $a_n$ and $frac 1n$
    $endgroup$
    – Omnomnomnom
    Jan 2 at 3:58










  • $begingroup$
    Why the downvote and the vote to close?
    $endgroup$
    – JavaMan
    Jan 2 at 4:19










5




5




$begingroup$
It is notable that $sqrt{a_n/n}$ is the geometric mean of $a_n$ and $frac 1n$
$endgroup$
– Omnomnomnom
Jan 2 at 3:58




$begingroup$
It is notable that $sqrt{a_n/n}$ is the geometric mean of $a_n$ and $frac 1n$
$endgroup$
– Omnomnomnom
Jan 2 at 3:58












$begingroup$
Why the downvote and the vote to close?
$endgroup$
– JavaMan
Jan 2 at 4:19






$begingroup$
Why the downvote and the vote to close?
$endgroup$
– JavaMan
Jan 2 at 4:19












3 Answers
3






active

oldest

votes


















6












$begingroup$

We can get by with assuming $sumlimits_nsqrt{a_n/n}$ converges, and the weaker condition that $a_n/n$ is decreasing.





Answering the Question



Let $u_n=sqrt{a_n/n}$, then $u_n$ is decreasing and $sumlimits_nu_n$ converges.
Since $u_n$ is decreasing,
$$
begin{align}
nu_n
&lesum_{k=1}^nu_ktag1\
&lesum_{k=1}^infty u_ktag2
end{align}
$$

Applying $(2)$ yields
$$
begin{align}
sum_{n=1}^infty nu_n^2
&leleft(sup_{1le nleinfty}nu_nright)sum_{n=1}^infty u_n\
&leleft(sum_{n=1}^infty u_nright)^2tag3
end{align}
$$

Note that $(3)$ is sharp if we consider the sequence
$$
u_n=left{begin{array}{}
1&text{if }n=1\
0&text{if }ngt1
end{array}right.
$$

Furthermore, $(3)$ says that $sumlimits_{n}a_n=sumlimits_{n}nu_n^2$ converges.





Stronger Inequality



Inequality $(3)$ answers the question, but we can get a bit stronger result.
$$
begin{align}
sum_{n=1}^infty nu_n^2
&lesum_{n=1}^inftysum_{k=1}^nu_ku_ntag4\
&=sum_{k=1}^inftysum_{n=k}^infty u_ku_ntag5\
&=frac12left[left(sum_{n=1}^infty u_nright)^2+sum_{n=1}^infty u_n^2right]tag6\
sum_{n=1}^infty(2n-1)u_n^2
&leleft(sum_{n=1}^infty u_nright)^2tag7
end{align}
$$

Explanation:
$(4)$: apply $(1)$
$(5)$: change order of summation
$(6)$: average $(4)$ and $(5)$
$(7)$: subtract $frac12sumlimits_{n=1}^infty u_n^2$ from both sides and double



Note that $(7)$ is sharp if we consider any of the sequences
$$
u_n=left{begin{array}{}
1&text{if }1le nle N\
0&text{if }ngt N
end{array}right.
$$






share|cite|improve this answer











$endgroup$













  • $begingroup$
    Letting $v_n= nu_n^2=(nu_n)u_n,$ we have, in the non-trivial case where every $u_n>0$, that $lim_{nto infty}v_n/u_n=0,$ so if $sum u_n$ converges then $sum v_n$ does too. ................+1
    $endgroup$
    – DanielWainfleet
    Jan 2 at 10:41



















16












$begingroup$

By the Cauchy condensation test, $sum_{n=1}^{infty} sqrt{a_n/n}$ converges if and only if



$$ sum_{n=1}^{infty} 2^nsqrt{a_{2^n}/2^n} = sum_{n=1}^{infty} sqrt{2^n a_{2^n}} $$



converges. Likewise, $sum_{n=1}^{infty} a_n$ converges if and only if $sum_{n=1}^{infty} 2^n a_{2^n}$ converges. Now the conclusion follows from the observation that, if $b_n geq 0$ and $sum_n b_n$ converges, then so does $sum_n b_n^2$.






share|cite|improve this answer









$endgroup$





















    4












    $begingroup$

    Let $b_n=sqrt{a_n}$. Then ${b_n}_{ngeq 1}$ is a decreasing sequence of positive real numbers and we want to show that
    $$ sum_{ngeq 1}frac{b_n}{sqrt{n}}<+inftyquadLongrightarrowquad sum_{ngeq 1}b_n^2 < +infty.$$
    By the Cauchy-Schwarz inequality disguised as Titu's lemma we have
    $$ sum_{n=N+1}^{2N}frac{b_n}{sqrt{n}} geq frac{left(sqrt{b_{N+1}}+ldots+sqrt{b_{2N}}right)^2}{sqrt{N+1}+ldots+sqrt{2N}}geq frac{N^2 b_{2N}}{frac{2}{3}(2sqrt{2}-1)Nsqrt{N}}geq frac{4}{5}sqrt{N},b_{2N} $$
    hence
    $$ sum_{kgeq 0}2^{k/2} b_{2^k} $$
    is convergent and $b_{2^k}=o(2^{-k/2})$. On the other hand
    $$begin{eqnarray*} sum_{n=N+1}^{2N}b_n^2 &=& sum_{n=N+1}^{2N}sqrt{n}b_ncdotfrac{b_n}{sqrt{n}}\&=&sqrt{2N} b_{2N}sum_{n=N+1}^{2N}frac{b_n}{sqrt{n}}+sum_{n=N+1}^{2N-1}left(frac{b_n}{sqrt{n}}-frac{b_{n+1}}{sqrt{n+1}}right)sum_{m=N+1}^{n}frac{b_n}{sqrt{n}}end{eqnarray*} $$
    by summation by parts, and assuming that $b_n$ is normalized in such a way that $sum_{ngeq 1}frac{b_n}{sqrt{n}}=1$, the RHS is bounded by
    $$begin{eqnarray*}&&sum_{n=N+1}^{2N-1}frac{b_n}{sqrt{n}}left[sqrt{2N} b_{2N}+sum_{n=N+1}^{2N-1}left(frac{b_n}{sqrt{n}}-frac{b_{n+1}}{sqrt{n+1}}right)right]\&=&sum_{n=N+1}^{2N-1}frac{b_n}{sqrt{n}}left[sqrt{2N} b_{2N}+frac{b_{N+1}}{sqrt{N+1}}-frac{b_{2N}}{sqrt{2N}}right]\&leq& sum_{n=N+1}^{2N-1}frac{b_n}{sqrt{n}}left[sqrt{2N} b_{2N}+frac{5sqrt{2}}{4N}right]\ &leq&frac{5sqrt{2}}{4}cdotfrac{N+2}{N+1}sum_{n=N+1}^{2N}frac{b_n}{sqrt{n}}.end{eqnarray*} $$
    This detour gives a quantitative improvement of the other proofs: by summing on $N=2^k$ with $kinmathbb{N}$, then getting rid of the normalization assumption, we get
    $$boxed{sum_{ngeq 1}b_n^2 leq b_1^2+color{blue}{frac{15sqrt{2}}{8}}left(sum_{ngeq 1}frac{b_n}{sqrt{n}}right)^2} $$
    and we may start wondering about the optimal constant that can replace $frac{15sqrt{2}}{8}$, like in Hardy's inequality. In this regard it makes sense to replace the short sums $sum_{n=N+1}^{2N}$ with $sum_{n=N+1}^{AN}$ and optimize on $A$. By considering $b_n=frac{1}{n^{1/2+varepsilon}}$, such that $sum_{ngeq 1}frac{b_n}{sqrt{n}}$ is just barely convergent, we have that the blue constant cannot be improved beyond $frac{7}{74}$.





    Alternative approach: if a sequence ${a_n}_{ngeq 1}$ is such that $sum_{ngeq 1}lambda_n a_n$ is finite for any ${lambda_n}_{ngeq 1}inell^2$, then ${a_n}_{ngeq 1}inell^2$. This is a consequence of the Banach-Steinhaus theorem, which can be proved independently by summation by parts (see page 150 of my notes). The constraints $0<a_{n+1}<a_n$ and $sum_{ngeq 1}frac{a_n}{sqrt{n}}=C<+infty$ should be more than enough to ensure that $sum_{ngeq 1}lambda_n a_n$ is finite for any $Lambdainell^2$, since $sum_{ngeq 1}frac{1}{n}$ is divergent.






    share|cite|improve this answer











    $endgroup$













    • $begingroup$
      This question is possibly related to your alternative approach.
      $endgroup$
      – robjohn
      Jan 3 at 21:10












    Your Answer








    StackExchange.ready(function() {
    var channelOptions = {
    tags: "".split(" "),
    id: "69"
    };
    initTagRenderer("".split(" "), "".split(" "), channelOptions);

    StackExchange.using("externalEditor", function() {
    // Have to fire editor after snippets, if snippets enabled
    if (StackExchange.settings.snippets.snippetsEnabled) {
    StackExchange.using("snippets", function() {
    createEditor();
    });
    }
    else {
    createEditor();
    }
    });

    function createEditor() {
    StackExchange.prepareEditor({
    heartbeatType: 'answer',
    autoActivateHeartbeat: false,
    convertImagesToLinks: true,
    noModals: true,
    showLowRepImageUploadWarning: true,
    reputationToPostImages: 10,
    bindNavPrevention: true,
    postfix: "",
    imageUploader: {
    brandingHtml: "Powered by u003ca class="icon-imgur-white" href="https://imgur.com/"u003eu003c/au003e",
    contentPolicyHtml: "User contributions licensed under u003ca href="https://creativecommons.org/licenses/by-sa/3.0/"u003ecc by-sa 3.0 with attribution requiredu003c/au003e u003ca href="https://stackoverflow.com/legal/content-policy"u003e(content policy)u003c/au003e",
    allowUrls: true
    },
    noCode: true, onDemand: true,
    discardSelector: ".discard-answer"
    ,immediatelyShowMarkdownHelp:true
    });


    }
    });














    draft saved

    draft discarded


















    StackExchange.ready(
    function () {
    StackExchange.openid.initPostLogin('.new-post-login', 'https%3a%2f%2fmath.stackexchange.com%2fquestions%2f3059124%2fsuppose-sum-n-1-infty-sqrta-n-n-is-convergent-prove-that-sum%23new-answer', 'question_page');
    }
    );

    Post as a guest















    Required, but never shown

























    3 Answers
    3






    active

    oldest

    votes








    3 Answers
    3






    active

    oldest

    votes









    active

    oldest

    votes






    active

    oldest

    votes









    6












    $begingroup$

    We can get by with assuming $sumlimits_nsqrt{a_n/n}$ converges, and the weaker condition that $a_n/n$ is decreasing.





    Answering the Question



    Let $u_n=sqrt{a_n/n}$, then $u_n$ is decreasing and $sumlimits_nu_n$ converges.
    Since $u_n$ is decreasing,
    $$
    begin{align}
    nu_n
    &lesum_{k=1}^nu_ktag1\
    &lesum_{k=1}^infty u_ktag2
    end{align}
    $$

    Applying $(2)$ yields
    $$
    begin{align}
    sum_{n=1}^infty nu_n^2
    &leleft(sup_{1le nleinfty}nu_nright)sum_{n=1}^infty u_n\
    &leleft(sum_{n=1}^infty u_nright)^2tag3
    end{align}
    $$

    Note that $(3)$ is sharp if we consider the sequence
    $$
    u_n=left{begin{array}{}
    1&text{if }n=1\
    0&text{if }ngt1
    end{array}right.
    $$

    Furthermore, $(3)$ says that $sumlimits_{n}a_n=sumlimits_{n}nu_n^2$ converges.





    Stronger Inequality



    Inequality $(3)$ answers the question, but we can get a bit stronger result.
    $$
    begin{align}
    sum_{n=1}^infty nu_n^2
    &lesum_{n=1}^inftysum_{k=1}^nu_ku_ntag4\
    &=sum_{k=1}^inftysum_{n=k}^infty u_ku_ntag5\
    &=frac12left[left(sum_{n=1}^infty u_nright)^2+sum_{n=1}^infty u_n^2right]tag6\
    sum_{n=1}^infty(2n-1)u_n^2
    &leleft(sum_{n=1}^infty u_nright)^2tag7
    end{align}
    $$

    Explanation:
    $(4)$: apply $(1)$
    $(5)$: change order of summation
    $(6)$: average $(4)$ and $(5)$
    $(7)$: subtract $frac12sumlimits_{n=1}^infty u_n^2$ from both sides and double



    Note that $(7)$ is sharp if we consider any of the sequences
    $$
    u_n=left{begin{array}{}
    1&text{if }1le nle N\
    0&text{if }ngt N
    end{array}right.
    $$






    share|cite|improve this answer











    $endgroup$













    • $begingroup$
      Letting $v_n= nu_n^2=(nu_n)u_n,$ we have, in the non-trivial case where every $u_n>0$, that $lim_{nto infty}v_n/u_n=0,$ so if $sum u_n$ converges then $sum v_n$ does too. ................+1
      $endgroup$
      – DanielWainfleet
      Jan 2 at 10:41
















    6












    $begingroup$

    We can get by with assuming $sumlimits_nsqrt{a_n/n}$ converges, and the weaker condition that $a_n/n$ is decreasing.





    Answering the Question



    Let $u_n=sqrt{a_n/n}$, then $u_n$ is decreasing and $sumlimits_nu_n$ converges.
    Since $u_n$ is decreasing,
    $$
    begin{align}
    nu_n
    &lesum_{k=1}^nu_ktag1\
    &lesum_{k=1}^infty u_ktag2
    end{align}
    $$

    Applying $(2)$ yields
    $$
    begin{align}
    sum_{n=1}^infty nu_n^2
    &leleft(sup_{1le nleinfty}nu_nright)sum_{n=1}^infty u_n\
    &leleft(sum_{n=1}^infty u_nright)^2tag3
    end{align}
    $$

    Note that $(3)$ is sharp if we consider the sequence
    $$
    u_n=left{begin{array}{}
    1&text{if }n=1\
    0&text{if }ngt1
    end{array}right.
    $$

    Furthermore, $(3)$ says that $sumlimits_{n}a_n=sumlimits_{n}nu_n^2$ converges.





    Stronger Inequality



    Inequality $(3)$ answers the question, but we can get a bit stronger result.
    $$
    begin{align}
    sum_{n=1}^infty nu_n^2
    &lesum_{n=1}^inftysum_{k=1}^nu_ku_ntag4\
    &=sum_{k=1}^inftysum_{n=k}^infty u_ku_ntag5\
    &=frac12left[left(sum_{n=1}^infty u_nright)^2+sum_{n=1}^infty u_n^2right]tag6\
    sum_{n=1}^infty(2n-1)u_n^2
    &leleft(sum_{n=1}^infty u_nright)^2tag7
    end{align}
    $$

    Explanation:
    $(4)$: apply $(1)$
    $(5)$: change order of summation
    $(6)$: average $(4)$ and $(5)$
    $(7)$: subtract $frac12sumlimits_{n=1}^infty u_n^2$ from both sides and double



    Note that $(7)$ is sharp if we consider any of the sequences
    $$
    u_n=left{begin{array}{}
    1&text{if }1le nle N\
    0&text{if }ngt N
    end{array}right.
    $$






    share|cite|improve this answer











    $endgroup$













    • $begingroup$
      Letting $v_n= nu_n^2=(nu_n)u_n,$ we have, in the non-trivial case where every $u_n>0$, that $lim_{nto infty}v_n/u_n=0,$ so if $sum u_n$ converges then $sum v_n$ does too. ................+1
      $endgroup$
      – DanielWainfleet
      Jan 2 at 10:41














    6












    6








    6





    $begingroup$

    We can get by with assuming $sumlimits_nsqrt{a_n/n}$ converges, and the weaker condition that $a_n/n$ is decreasing.





    Answering the Question



    Let $u_n=sqrt{a_n/n}$, then $u_n$ is decreasing and $sumlimits_nu_n$ converges.
    Since $u_n$ is decreasing,
    $$
    begin{align}
    nu_n
    &lesum_{k=1}^nu_ktag1\
    &lesum_{k=1}^infty u_ktag2
    end{align}
    $$

    Applying $(2)$ yields
    $$
    begin{align}
    sum_{n=1}^infty nu_n^2
    &leleft(sup_{1le nleinfty}nu_nright)sum_{n=1}^infty u_n\
    &leleft(sum_{n=1}^infty u_nright)^2tag3
    end{align}
    $$

    Note that $(3)$ is sharp if we consider the sequence
    $$
    u_n=left{begin{array}{}
    1&text{if }n=1\
    0&text{if }ngt1
    end{array}right.
    $$

    Furthermore, $(3)$ says that $sumlimits_{n}a_n=sumlimits_{n}nu_n^2$ converges.





    Stronger Inequality



    Inequality $(3)$ answers the question, but we can get a bit stronger result.
    $$
    begin{align}
    sum_{n=1}^infty nu_n^2
    &lesum_{n=1}^inftysum_{k=1}^nu_ku_ntag4\
    &=sum_{k=1}^inftysum_{n=k}^infty u_ku_ntag5\
    &=frac12left[left(sum_{n=1}^infty u_nright)^2+sum_{n=1}^infty u_n^2right]tag6\
    sum_{n=1}^infty(2n-1)u_n^2
    &leleft(sum_{n=1}^infty u_nright)^2tag7
    end{align}
    $$

    Explanation:
    $(4)$: apply $(1)$
    $(5)$: change order of summation
    $(6)$: average $(4)$ and $(5)$
    $(7)$: subtract $frac12sumlimits_{n=1}^infty u_n^2$ from both sides and double



    Note that $(7)$ is sharp if we consider any of the sequences
    $$
    u_n=left{begin{array}{}
    1&text{if }1le nle N\
    0&text{if }ngt N
    end{array}right.
    $$






    share|cite|improve this answer











    $endgroup$



    We can get by with assuming $sumlimits_nsqrt{a_n/n}$ converges, and the weaker condition that $a_n/n$ is decreasing.





    Answering the Question



    Let $u_n=sqrt{a_n/n}$, then $u_n$ is decreasing and $sumlimits_nu_n$ converges.
    Since $u_n$ is decreasing,
    $$
    begin{align}
    nu_n
    &lesum_{k=1}^nu_ktag1\
    &lesum_{k=1}^infty u_ktag2
    end{align}
    $$

    Applying $(2)$ yields
    $$
    begin{align}
    sum_{n=1}^infty nu_n^2
    &leleft(sup_{1le nleinfty}nu_nright)sum_{n=1}^infty u_n\
    &leleft(sum_{n=1}^infty u_nright)^2tag3
    end{align}
    $$

    Note that $(3)$ is sharp if we consider the sequence
    $$
    u_n=left{begin{array}{}
    1&text{if }n=1\
    0&text{if }ngt1
    end{array}right.
    $$

    Furthermore, $(3)$ says that $sumlimits_{n}a_n=sumlimits_{n}nu_n^2$ converges.





    Stronger Inequality



    Inequality $(3)$ answers the question, but we can get a bit stronger result.
    $$
    begin{align}
    sum_{n=1}^infty nu_n^2
    &lesum_{n=1}^inftysum_{k=1}^nu_ku_ntag4\
    &=sum_{k=1}^inftysum_{n=k}^infty u_ku_ntag5\
    &=frac12left[left(sum_{n=1}^infty u_nright)^2+sum_{n=1}^infty u_n^2right]tag6\
    sum_{n=1}^infty(2n-1)u_n^2
    &leleft(sum_{n=1}^infty u_nright)^2tag7
    end{align}
    $$

    Explanation:
    $(4)$: apply $(1)$
    $(5)$: change order of summation
    $(6)$: average $(4)$ and $(5)$
    $(7)$: subtract $frac12sumlimits_{n=1}^infty u_n^2$ from both sides and double



    Note that $(7)$ is sharp if we consider any of the sequences
    $$
    u_n=left{begin{array}{}
    1&text{if }1le nle N\
    0&text{if }ngt N
    end{array}right.
    $$







    share|cite|improve this answer














    share|cite|improve this answer



    share|cite|improve this answer








    edited Jan 3 at 9:49

























    answered Jan 2 at 8:41









    robjohnrobjohn

    271k27316643




    271k27316643












    • $begingroup$
      Letting $v_n= nu_n^2=(nu_n)u_n,$ we have, in the non-trivial case where every $u_n>0$, that $lim_{nto infty}v_n/u_n=0,$ so if $sum u_n$ converges then $sum v_n$ does too. ................+1
      $endgroup$
      – DanielWainfleet
      Jan 2 at 10:41


















    • $begingroup$
      Letting $v_n= nu_n^2=(nu_n)u_n,$ we have, in the non-trivial case where every $u_n>0$, that $lim_{nto infty}v_n/u_n=0,$ so if $sum u_n$ converges then $sum v_n$ does too. ................+1
      $endgroup$
      – DanielWainfleet
      Jan 2 at 10:41
















    $begingroup$
    Letting $v_n= nu_n^2=(nu_n)u_n,$ we have, in the non-trivial case where every $u_n>0$, that $lim_{nto infty}v_n/u_n=0,$ so if $sum u_n$ converges then $sum v_n$ does too. ................+1
    $endgroup$
    – DanielWainfleet
    Jan 2 at 10:41




    $begingroup$
    Letting $v_n= nu_n^2=(nu_n)u_n,$ we have, in the non-trivial case where every $u_n>0$, that $lim_{nto infty}v_n/u_n=0,$ so if $sum u_n$ converges then $sum v_n$ does too. ................+1
    $endgroup$
    – DanielWainfleet
    Jan 2 at 10:41











    16












    $begingroup$

    By the Cauchy condensation test, $sum_{n=1}^{infty} sqrt{a_n/n}$ converges if and only if



    $$ sum_{n=1}^{infty} 2^nsqrt{a_{2^n}/2^n} = sum_{n=1}^{infty} sqrt{2^n a_{2^n}} $$



    converges. Likewise, $sum_{n=1}^{infty} a_n$ converges if and only if $sum_{n=1}^{infty} 2^n a_{2^n}$ converges. Now the conclusion follows from the observation that, if $b_n geq 0$ and $sum_n b_n$ converges, then so does $sum_n b_n^2$.






    share|cite|improve this answer









    $endgroup$


















      16












      $begingroup$

      By the Cauchy condensation test, $sum_{n=1}^{infty} sqrt{a_n/n}$ converges if and only if



      $$ sum_{n=1}^{infty} 2^nsqrt{a_{2^n}/2^n} = sum_{n=1}^{infty} sqrt{2^n a_{2^n}} $$



      converges. Likewise, $sum_{n=1}^{infty} a_n$ converges if and only if $sum_{n=1}^{infty} 2^n a_{2^n}$ converges. Now the conclusion follows from the observation that, if $b_n geq 0$ and $sum_n b_n$ converges, then so does $sum_n b_n^2$.






      share|cite|improve this answer









      $endgroup$
















        16












        16








        16





        $begingroup$

        By the Cauchy condensation test, $sum_{n=1}^{infty} sqrt{a_n/n}$ converges if and only if



        $$ sum_{n=1}^{infty} 2^nsqrt{a_{2^n}/2^n} = sum_{n=1}^{infty} sqrt{2^n a_{2^n}} $$



        converges. Likewise, $sum_{n=1}^{infty} a_n$ converges if and only if $sum_{n=1}^{infty} 2^n a_{2^n}$ converges. Now the conclusion follows from the observation that, if $b_n geq 0$ and $sum_n b_n$ converges, then so does $sum_n b_n^2$.






        share|cite|improve this answer









        $endgroup$



        By the Cauchy condensation test, $sum_{n=1}^{infty} sqrt{a_n/n}$ converges if and only if



        $$ sum_{n=1}^{infty} 2^nsqrt{a_{2^n}/2^n} = sum_{n=1}^{infty} sqrt{2^n a_{2^n}} $$



        converges. Likewise, $sum_{n=1}^{infty} a_n$ converges if and only if $sum_{n=1}^{infty} 2^n a_{2^n}$ converges. Now the conclusion follows from the observation that, if $b_n geq 0$ and $sum_n b_n$ converges, then so does $sum_n b_n^2$.







        share|cite|improve this answer












        share|cite|improve this answer



        share|cite|improve this answer










        answered Jan 2 at 4:16









        Sangchul LeeSangchul Lee

        96.8k12173283




        96.8k12173283























            4












            $begingroup$

            Let $b_n=sqrt{a_n}$. Then ${b_n}_{ngeq 1}$ is a decreasing sequence of positive real numbers and we want to show that
            $$ sum_{ngeq 1}frac{b_n}{sqrt{n}}<+inftyquadLongrightarrowquad sum_{ngeq 1}b_n^2 < +infty.$$
            By the Cauchy-Schwarz inequality disguised as Titu's lemma we have
            $$ sum_{n=N+1}^{2N}frac{b_n}{sqrt{n}} geq frac{left(sqrt{b_{N+1}}+ldots+sqrt{b_{2N}}right)^2}{sqrt{N+1}+ldots+sqrt{2N}}geq frac{N^2 b_{2N}}{frac{2}{3}(2sqrt{2}-1)Nsqrt{N}}geq frac{4}{5}sqrt{N},b_{2N} $$
            hence
            $$ sum_{kgeq 0}2^{k/2} b_{2^k} $$
            is convergent and $b_{2^k}=o(2^{-k/2})$. On the other hand
            $$begin{eqnarray*} sum_{n=N+1}^{2N}b_n^2 &=& sum_{n=N+1}^{2N}sqrt{n}b_ncdotfrac{b_n}{sqrt{n}}\&=&sqrt{2N} b_{2N}sum_{n=N+1}^{2N}frac{b_n}{sqrt{n}}+sum_{n=N+1}^{2N-1}left(frac{b_n}{sqrt{n}}-frac{b_{n+1}}{sqrt{n+1}}right)sum_{m=N+1}^{n}frac{b_n}{sqrt{n}}end{eqnarray*} $$
            by summation by parts, and assuming that $b_n$ is normalized in such a way that $sum_{ngeq 1}frac{b_n}{sqrt{n}}=1$, the RHS is bounded by
            $$begin{eqnarray*}&&sum_{n=N+1}^{2N-1}frac{b_n}{sqrt{n}}left[sqrt{2N} b_{2N}+sum_{n=N+1}^{2N-1}left(frac{b_n}{sqrt{n}}-frac{b_{n+1}}{sqrt{n+1}}right)right]\&=&sum_{n=N+1}^{2N-1}frac{b_n}{sqrt{n}}left[sqrt{2N} b_{2N}+frac{b_{N+1}}{sqrt{N+1}}-frac{b_{2N}}{sqrt{2N}}right]\&leq& sum_{n=N+1}^{2N-1}frac{b_n}{sqrt{n}}left[sqrt{2N} b_{2N}+frac{5sqrt{2}}{4N}right]\ &leq&frac{5sqrt{2}}{4}cdotfrac{N+2}{N+1}sum_{n=N+1}^{2N}frac{b_n}{sqrt{n}}.end{eqnarray*} $$
            This detour gives a quantitative improvement of the other proofs: by summing on $N=2^k$ with $kinmathbb{N}$, then getting rid of the normalization assumption, we get
            $$boxed{sum_{ngeq 1}b_n^2 leq b_1^2+color{blue}{frac{15sqrt{2}}{8}}left(sum_{ngeq 1}frac{b_n}{sqrt{n}}right)^2} $$
            and we may start wondering about the optimal constant that can replace $frac{15sqrt{2}}{8}$, like in Hardy's inequality. In this regard it makes sense to replace the short sums $sum_{n=N+1}^{2N}$ with $sum_{n=N+1}^{AN}$ and optimize on $A$. By considering $b_n=frac{1}{n^{1/2+varepsilon}}$, such that $sum_{ngeq 1}frac{b_n}{sqrt{n}}$ is just barely convergent, we have that the blue constant cannot be improved beyond $frac{7}{74}$.





            Alternative approach: if a sequence ${a_n}_{ngeq 1}$ is such that $sum_{ngeq 1}lambda_n a_n$ is finite for any ${lambda_n}_{ngeq 1}inell^2$, then ${a_n}_{ngeq 1}inell^2$. This is a consequence of the Banach-Steinhaus theorem, which can be proved independently by summation by parts (see page 150 of my notes). The constraints $0<a_{n+1}<a_n$ and $sum_{ngeq 1}frac{a_n}{sqrt{n}}=C<+infty$ should be more than enough to ensure that $sum_{ngeq 1}lambda_n a_n$ is finite for any $Lambdainell^2$, since $sum_{ngeq 1}frac{1}{n}$ is divergent.






            share|cite|improve this answer











            $endgroup$













            • $begingroup$
              This question is possibly related to your alternative approach.
              $endgroup$
              – robjohn
              Jan 3 at 21:10
















            4












            $begingroup$

            Let $b_n=sqrt{a_n}$. Then ${b_n}_{ngeq 1}$ is a decreasing sequence of positive real numbers and we want to show that
            $$ sum_{ngeq 1}frac{b_n}{sqrt{n}}<+inftyquadLongrightarrowquad sum_{ngeq 1}b_n^2 < +infty.$$
            By the Cauchy-Schwarz inequality disguised as Titu's lemma we have
            $$ sum_{n=N+1}^{2N}frac{b_n}{sqrt{n}} geq frac{left(sqrt{b_{N+1}}+ldots+sqrt{b_{2N}}right)^2}{sqrt{N+1}+ldots+sqrt{2N}}geq frac{N^2 b_{2N}}{frac{2}{3}(2sqrt{2}-1)Nsqrt{N}}geq frac{4}{5}sqrt{N},b_{2N} $$
            hence
            $$ sum_{kgeq 0}2^{k/2} b_{2^k} $$
            is convergent and $b_{2^k}=o(2^{-k/2})$. On the other hand
            $$begin{eqnarray*} sum_{n=N+1}^{2N}b_n^2 &=& sum_{n=N+1}^{2N}sqrt{n}b_ncdotfrac{b_n}{sqrt{n}}\&=&sqrt{2N} b_{2N}sum_{n=N+1}^{2N}frac{b_n}{sqrt{n}}+sum_{n=N+1}^{2N-1}left(frac{b_n}{sqrt{n}}-frac{b_{n+1}}{sqrt{n+1}}right)sum_{m=N+1}^{n}frac{b_n}{sqrt{n}}end{eqnarray*} $$
            by summation by parts, and assuming that $b_n$ is normalized in such a way that $sum_{ngeq 1}frac{b_n}{sqrt{n}}=1$, the RHS is bounded by
            $$begin{eqnarray*}&&sum_{n=N+1}^{2N-1}frac{b_n}{sqrt{n}}left[sqrt{2N} b_{2N}+sum_{n=N+1}^{2N-1}left(frac{b_n}{sqrt{n}}-frac{b_{n+1}}{sqrt{n+1}}right)right]\&=&sum_{n=N+1}^{2N-1}frac{b_n}{sqrt{n}}left[sqrt{2N} b_{2N}+frac{b_{N+1}}{sqrt{N+1}}-frac{b_{2N}}{sqrt{2N}}right]\&leq& sum_{n=N+1}^{2N-1}frac{b_n}{sqrt{n}}left[sqrt{2N} b_{2N}+frac{5sqrt{2}}{4N}right]\ &leq&frac{5sqrt{2}}{4}cdotfrac{N+2}{N+1}sum_{n=N+1}^{2N}frac{b_n}{sqrt{n}}.end{eqnarray*} $$
            This detour gives a quantitative improvement of the other proofs: by summing on $N=2^k$ with $kinmathbb{N}$, then getting rid of the normalization assumption, we get
            $$boxed{sum_{ngeq 1}b_n^2 leq b_1^2+color{blue}{frac{15sqrt{2}}{8}}left(sum_{ngeq 1}frac{b_n}{sqrt{n}}right)^2} $$
            and we may start wondering about the optimal constant that can replace $frac{15sqrt{2}}{8}$, like in Hardy's inequality. In this regard it makes sense to replace the short sums $sum_{n=N+1}^{2N}$ with $sum_{n=N+1}^{AN}$ and optimize on $A$. By considering $b_n=frac{1}{n^{1/2+varepsilon}}$, such that $sum_{ngeq 1}frac{b_n}{sqrt{n}}$ is just barely convergent, we have that the blue constant cannot be improved beyond $frac{7}{74}$.





            Alternative approach: if a sequence ${a_n}_{ngeq 1}$ is such that $sum_{ngeq 1}lambda_n a_n$ is finite for any ${lambda_n}_{ngeq 1}inell^2$, then ${a_n}_{ngeq 1}inell^2$. This is a consequence of the Banach-Steinhaus theorem, which can be proved independently by summation by parts (see page 150 of my notes). The constraints $0<a_{n+1}<a_n$ and $sum_{ngeq 1}frac{a_n}{sqrt{n}}=C<+infty$ should be more than enough to ensure that $sum_{ngeq 1}lambda_n a_n$ is finite for any $Lambdainell^2$, since $sum_{ngeq 1}frac{1}{n}$ is divergent.






            share|cite|improve this answer











            $endgroup$













            • $begingroup$
              This question is possibly related to your alternative approach.
              $endgroup$
              – robjohn
              Jan 3 at 21:10














            4












            4








            4





            $begingroup$

            Let $b_n=sqrt{a_n}$. Then ${b_n}_{ngeq 1}$ is a decreasing sequence of positive real numbers and we want to show that
            $$ sum_{ngeq 1}frac{b_n}{sqrt{n}}<+inftyquadLongrightarrowquad sum_{ngeq 1}b_n^2 < +infty.$$
            By the Cauchy-Schwarz inequality disguised as Titu's lemma we have
            $$ sum_{n=N+1}^{2N}frac{b_n}{sqrt{n}} geq frac{left(sqrt{b_{N+1}}+ldots+sqrt{b_{2N}}right)^2}{sqrt{N+1}+ldots+sqrt{2N}}geq frac{N^2 b_{2N}}{frac{2}{3}(2sqrt{2}-1)Nsqrt{N}}geq frac{4}{5}sqrt{N},b_{2N} $$
            hence
            $$ sum_{kgeq 0}2^{k/2} b_{2^k} $$
            is convergent and $b_{2^k}=o(2^{-k/2})$. On the other hand
            $$begin{eqnarray*} sum_{n=N+1}^{2N}b_n^2 &=& sum_{n=N+1}^{2N}sqrt{n}b_ncdotfrac{b_n}{sqrt{n}}\&=&sqrt{2N} b_{2N}sum_{n=N+1}^{2N}frac{b_n}{sqrt{n}}+sum_{n=N+1}^{2N-1}left(frac{b_n}{sqrt{n}}-frac{b_{n+1}}{sqrt{n+1}}right)sum_{m=N+1}^{n}frac{b_n}{sqrt{n}}end{eqnarray*} $$
            by summation by parts, and assuming that $b_n$ is normalized in such a way that $sum_{ngeq 1}frac{b_n}{sqrt{n}}=1$, the RHS is bounded by
            $$begin{eqnarray*}&&sum_{n=N+1}^{2N-1}frac{b_n}{sqrt{n}}left[sqrt{2N} b_{2N}+sum_{n=N+1}^{2N-1}left(frac{b_n}{sqrt{n}}-frac{b_{n+1}}{sqrt{n+1}}right)right]\&=&sum_{n=N+1}^{2N-1}frac{b_n}{sqrt{n}}left[sqrt{2N} b_{2N}+frac{b_{N+1}}{sqrt{N+1}}-frac{b_{2N}}{sqrt{2N}}right]\&leq& sum_{n=N+1}^{2N-1}frac{b_n}{sqrt{n}}left[sqrt{2N} b_{2N}+frac{5sqrt{2}}{4N}right]\ &leq&frac{5sqrt{2}}{4}cdotfrac{N+2}{N+1}sum_{n=N+1}^{2N}frac{b_n}{sqrt{n}}.end{eqnarray*} $$
            This detour gives a quantitative improvement of the other proofs: by summing on $N=2^k$ with $kinmathbb{N}$, then getting rid of the normalization assumption, we get
            $$boxed{sum_{ngeq 1}b_n^2 leq b_1^2+color{blue}{frac{15sqrt{2}}{8}}left(sum_{ngeq 1}frac{b_n}{sqrt{n}}right)^2} $$
            and we may start wondering about the optimal constant that can replace $frac{15sqrt{2}}{8}$, like in Hardy's inequality. In this regard it makes sense to replace the short sums $sum_{n=N+1}^{2N}$ with $sum_{n=N+1}^{AN}$ and optimize on $A$. By considering $b_n=frac{1}{n^{1/2+varepsilon}}$, such that $sum_{ngeq 1}frac{b_n}{sqrt{n}}$ is just barely convergent, we have that the blue constant cannot be improved beyond $frac{7}{74}$.





            Alternative approach: if a sequence ${a_n}_{ngeq 1}$ is such that $sum_{ngeq 1}lambda_n a_n$ is finite for any ${lambda_n}_{ngeq 1}inell^2$, then ${a_n}_{ngeq 1}inell^2$. This is a consequence of the Banach-Steinhaus theorem, which can be proved independently by summation by parts (see page 150 of my notes). The constraints $0<a_{n+1}<a_n$ and $sum_{ngeq 1}frac{a_n}{sqrt{n}}=C<+infty$ should be more than enough to ensure that $sum_{ngeq 1}lambda_n a_n$ is finite for any $Lambdainell^2$, since $sum_{ngeq 1}frac{1}{n}$ is divergent.






            share|cite|improve this answer











            $endgroup$



            Let $b_n=sqrt{a_n}$. Then ${b_n}_{ngeq 1}$ is a decreasing sequence of positive real numbers and we want to show that
            $$ sum_{ngeq 1}frac{b_n}{sqrt{n}}<+inftyquadLongrightarrowquad sum_{ngeq 1}b_n^2 < +infty.$$
            By the Cauchy-Schwarz inequality disguised as Titu's lemma we have
            $$ sum_{n=N+1}^{2N}frac{b_n}{sqrt{n}} geq frac{left(sqrt{b_{N+1}}+ldots+sqrt{b_{2N}}right)^2}{sqrt{N+1}+ldots+sqrt{2N}}geq frac{N^2 b_{2N}}{frac{2}{3}(2sqrt{2}-1)Nsqrt{N}}geq frac{4}{5}sqrt{N},b_{2N} $$
            hence
            $$ sum_{kgeq 0}2^{k/2} b_{2^k} $$
            is convergent and $b_{2^k}=o(2^{-k/2})$. On the other hand
            $$begin{eqnarray*} sum_{n=N+1}^{2N}b_n^2 &=& sum_{n=N+1}^{2N}sqrt{n}b_ncdotfrac{b_n}{sqrt{n}}\&=&sqrt{2N} b_{2N}sum_{n=N+1}^{2N}frac{b_n}{sqrt{n}}+sum_{n=N+1}^{2N-1}left(frac{b_n}{sqrt{n}}-frac{b_{n+1}}{sqrt{n+1}}right)sum_{m=N+1}^{n}frac{b_n}{sqrt{n}}end{eqnarray*} $$
            by summation by parts, and assuming that $b_n$ is normalized in such a way that $sum_{ngeq 1}frac{b_n}{sqrt{n}}=1$, the RHS is bounded by
            $$begin{eqnarray*}&&sum_{n=N+1}^{2N-1}frac{b_n}{sqrt{n}}left[sqrt{2N} b_{2N}+sum_{n=N+1}^{2N-1}left(frac{b_n}{sqrt{n}}-frac{b_{n+1}}{sqrt{n+1}}right)right]\&=&sum_{n=N+1}^{2N-1}frac{b_n}{sqrt{n}}left[sqrt{2N} b_{2N}+frac{b_{N+1}}{sqrt{N+1}}-frac{b_{2N}}{sqrt{2N}}right]\&leq& sum_{n=N+1}^{2N-1}frac{b_n}{sqrt{n}}left[sqrt{2N} b_{2N}+frac{5sqrt{2}}{4N}right]\ &leq&frac{5sqrt{2}}{4}cdotfrac{N+2}{N+1}sum_{n=N+1}^{2N}frac{b_n}{sqrt{n}}.end{eqnarray*} $$
            This detour gives a quantitative improvement of the other proofs: by summing on $N=2^k$ with $kinmathbb{N}$, then getting rid of the normalization assumption, we get
            $$boxed{sum_{ngeq 1}b_n^2 leq b_1^2+color{blue}{frac{15sqrt{2}}{8}}left(sum_{ngeq 1}frac{b_n}{sqrt{n}}right)^2} $$
            and we may start wondering about the optimal constant that can replace $frac{15sqrt{2}}{8}$, like in Hardy's inequality. In this regard it makes sense to replace the short sums $sum_{n=N+1}^{2N}$ with $sum_{n=N+1}^{AN}$ and optimize on $A$. By considering $b_n=frac{1}{n^{1/2+varepsilon}}$, such that $sum_{ngeq 1}frac{b_n}{sqrt{n}}$ is just barely convergent, we have that the blue constant cannot be improved beyond $frac{7}{74}$.





            Alternative approach: if a sequence ${a_n}_{ngeq 1}$ is such that $sum_{ngeq 1}lambda_n a_n$ is finite for any ${lambda_n}_{ngeq 1}inell^2$, then ${a_n}_{ngeq 1}inell^2$. This is a consequence of the Banach-Steinhaus theorem, which can be proved independently by summation by parts (see page 150 of my notes). The constraints $0<a_{n+1}<a_n$ and $sum_{ngeq 1}frac{a_n}{sqrt{n}}=C<+infty$ should be more than enough to ensure that $sum_{ngeq 1}lambda_n a_n$ is finite for any $Lambdainell^2$, since $sum_{ngeq 1}frac{1}{n}$ is divergent.







            share|cite|improve this answer














            share|cite|improve this answer



            share|cite|improve this answer








            edited Jan 3 at 20:13

























            answered Jan 2 at 9:48









            Jack D'AurizioJack D'Aurizio

            292k33284674




            292k33284674












            • $begingroup$
              This question is possibly related to your alternative approach.
              $endgroup$
              – robjohn
              Jan 3 at 21:10


















            • $begingroup$
              This question is possibly related to your alternative approach.
              $endgroup$
              – robjohn
              Jan 3 at 21:10
















            $begingroup$
            This question is possibly related to your alternative approach.
            $endgroup$
            – robjohn
            Jan 3 at 21:10




            $begingroup$
            This question is possibly related to your alternative approach.
            $endgroup$
            – robjohn
            Jan 3 at 21:10


















            draft saved

            draft discarded




















































            Thanks for contributing an answer to Mathematics Stack Exchange!


            • Please be sure to answer the question. Provide details and share your research!

            But avoid



            • Asking for help, clarification, or responding to other answers.

            • Making statements based on opinion; back them up with references or personal experience.


            Use MathJax to format equations. MathJax reference.


            To learn more, see our tips on writing great answers.




            draft saved


            draft discarded














            StackExchange.ready(
            function () {
            StackExchange.openid.initPostLogin('.new-post-login', 'https%3a%2f%2fmath.stackexchange.com%2fquestions%2f3059124%2fsuppose-sum-n-1-infty-sqrta-n-n-is-convergent-prove-that-sum%23new-answer', 'question_page');
            }
            );

            Post as a guest















            Required, but never shown





















































            Required, but never shown














            Required, but never shown












            Required, but never shown







            Required, but never shown

































            Required, but never shown














            Required, but never shown












            Required, but never shown







            Required, but never shown







            Popular posts from this blog

            Biblatex bibliography style without URLs when DOI exists (in Overleaf with Zotero bibliography)

            ComboBox Display Member on multiple fields

            Is it possible to collect Nectar points via Trainline?